Đến nội dung

Hình ảnh

[IMO 2012 - P.2] Chứng minh: \[(1+a_2)^2(1+a_3)^3\cdots (1+a_n)^n > n^n\]

- - - - -

  • Please log in to reply
Chủ đề này có 2 trả lời

#1
Crystal

Crystal

    ANGRY BIRDS

  • Hiệp sỹ
  • 5534 Bài viết
Problem: Let $n \ge 3$ be an integer, and let $a_2, a_3, \ldots , a_n$ be positive real numbers such that $a_2\cdots a_n = 1.$ Prove that \[(1+a_2)^2(1+a_3)^3\cdots (1+a_n)^n > n^n\]

Bài toán: Cho số nguyên $n \ge 3$ và ${a_2},{a_3},...,{a_n}$ là các số thực dương thỏa mãn ${a_2}{a_3}...{a_n} = 1$. Chứng minh rằng:
\[(1+a_2)^2(1+a_3)^3\cdots (1+a_n)^n > n^n\]

#2
Tham Lang

Tham Lang

    Thượng úy

  • Thành viên
  • 1149 Bài viết
Hi hi, đúng là dễ hơn đề thi ĐH thật :D
Áp dụng $AM-GM$ ta có ngay :
$$(1+a_k)^k=\left (\dfrac{1}{k-1}.(k-1)+a_k\right )^k\ge \left (\dfrac{k\sqrt[k]{a_k}}{\sqrt[k]{(k-1)^{k-1}}}\right )^k=\dfrac{a_k}{(k-1)^{k-1}}.k^k$$
Nên do đó :
$$(1+a_2)^2(1+a_3)^3...(1+a_n)^n\ge n^n.a_2...a_n=n^n$$
Nhưng dấu "=" không xảy ra, suy ra ĐPCM.
_____________________________________________
@hxthanh: :D Tại bài này quá lỏng thôi!

Off vĩnh viễn ! Không ngày trở lại.......


#3
namcpnh

namcpnh

    Red Devil

  • Hiệp sỹ
  • 1153 Bài viết

Hi hi, đúng là dễ hơn đề thi ĐH thật :D
Áp dụng $AM-GM$ ta có ngay :
$$(1+a_k)^k=\left (\dfrac{1}{k-1}.(k-1)+a_k\right )^k\ge \left (\dfrac{k\sqrt[k]{a_k}}{\sqrt[k]{(k-1)^{k-1}}}\right )^k=\dfrac{a_k}{(k-1)^{k-1}}.k^k$$
Nên do đó :
$$(1+a_2)^2(1+a_3)^3...(1+a_n)^n\ge n^n.a_2...a_n=n^n$$
Nhưng dấu "=" không xảy ra, suy ra ĐPCM.

Sao đề năm nay dễ vậy?Câu 1 và câu 2 làm có mấy dòng là ra.Chắc là làm 2 câu này trong vòng 30' còn 4 tiếng suy nghĩ câu 3. :icon6:

Cùng chung sức làm chuyên đề hay cho diễn đàn tại :

Dãy số-giới hạn, Đa thức , Hình học , Phương trình hàm , PT-HPT-BPT , Số học.

Wolframalpha đây





1 người đang xem chủ đề

0 thành viên, 1 khách, 0 thành viên ẩn danh